Physics
Physics
1st Edition
Walker
ISBN: 9780133256925
Textbook solutions

All Solutions

Page 738: Assessment

Exercise 53
Step 1
1 of 2
The answer is NO. It depends on the initial conditions of the proton’s motion. A proton in a region of constant electric field experiences a constant electric force, which causes the proton to accelerate in the direction of the electric field. But maybe the proton has an initial perpendicular component of velocity, in which case it starts rotating. Or maybe the proton is initially moving in the opposite direction of the electric field, in such case, the proton slows down before it eventually stops and turns to move in the direction of the electric field.{}
Result
2 of 2
no; it depends on the initial velocity of the proton{}
Exercise 54
Step 1
1 of 2
The answer is YES. A proton in a region of constant electric field experiences a constant force in the direction of the electric field. We know from Newton’s second law that the direction of acceleration is the same as the applied force, in which case the proton’s acceleration is parallel to the electric field.{}
Result
2 of 2
yes; the acceleration is parallel to the electric force, which is in the direction of the electric field in case of a proton.{}
Exercise 55
Step 1
1 of 2
Charging by induction is a way to charge an object without touching it. In charging by induction, there is no actual transfer of charges between the charging source and the induced object. Charging by induction depends on the concept of electrical polarization to separate between the positive and negative charges and grounding to get rid of the charge repelled by the charge on the charging source. The leftover (induced) charge is distributed symmetrically on the induced object, and it has the opposite sign of the charge on the charging source.

In contrast, charging by contact involves an actual transfer of charges between the charging source and the object to charge. Moreover, the charged object acquires a charge with the same sign as the charge on the charging source.

Result
2 of 2
Charging by induction involves no transfer of charges and produces a charged object with a charge having the opposite sign of the charge on the charging source, whereas charging by contact involves transfer of charges and produces a charged object with a charge having the same sign as the source charge.{}
Exercise 56
Step 1
1 of 2
The answer depends on the separation $d$. Far away from the system of charges (the separation $d$ is negligible compared to the distance between the charges and the location at which the total electric field is considered), the two charges appear as if they are on top of each other with zero total charge and zero total electric field. In contrast, for a point closely separated from the charges, for which $d$ is comparable to the distance of the charges to location of the point, the total electric field is nonzero, as the electric field due to the closer charge overcomes that due to the further charge.
Result
2 of 2
It depends on the separation $d$. If the point of interest is very far from the charges, the total electric field is practically zero. If the point of interest is close enough from the charges, the electric fields due to the charges don’t completely cancel and we have a nonzero electric field.{}
Exercise 57
Step 1
1 of 2
The answer is No. Two opposite charges with unequal magnitudes in a region of uniform electric field will experience forces different in magnitude and direction. That is because the electric force is related to the electric field by $vec{pmb F} = qvec{pmb E}$, which elucidates the electric force depends on the magnitude and sign of the charge experiencing the force (even though $vec{pmb E}$ is uniform).
Result
2 of 2
no; the electric force on a charge depends on the magnitude and sign of this charge in addition to the electric field at the location of this charge{}
Exercise 58
Step 1
1 of 2
The charge is the source of electric field. So, once you have a charge, an associated electric field is developed everywhere around it, even in vacuum. The fact that light (which is basically a disturbance in electric and magnetic field) propagates from the sun to the earth through vacuum would $not$ be possible if the electric field doesn’t exist in a vacuum.{}
Result
2 of 2
yes; an electric field originates from a charge and exists everywhere around it, including in a vacuum{}
Exercise 59
Step 1
1 of 2
By definition, electric field lines always point in the direction of the electric force. Since the electric force, and hence the electric field, can point only in one direction at any given location, it follows that the field lines cannot cross. If they did, the field at the intersection point would have two conflicting directions.{}
Result
2 of 2
the electric force (or the electric field) must have a definite direction at any given location{}
Exercise 60
Step 1
1 of 2
The magnitude of the electric field can be calculated using $F = |q|E$. Solving for $E$ and substituting the know values, we find

$$
E = frac{F}{|q|} = frac{0.80;mathrm{N}}{3.6times10^{-6};mathrm{C}} = 2.2times10^5;mathrm{N/C}
$$

Result
2 of 2
$2.2times10^5$ N/C
Exercise 61
Step 1
1 of 2
Since the force is opposite in direction to the electric field, it follows that the charge has a negative sign. The magnitude of the charge can be found using $F = |q|E$. Solving for $|q|$ and substituting the known values, we find

$$
|q| = frac{F}{E} = frac{0.61;mathrm{N}}{4.7times10^4;mathrm{N/C}} = 13;mumathrm{C}
$$

We see the charge on the object is $-13;mu$C.

Result
2 of 2
$-13;mu$C
Exercise 62
Step 1
1 of 2
As we assume all the other forces on the object can be ignored, the object accelerates due to the electric force only. Applying Newton’s second law, we have

$$
F = ma
$$

where $F$ is the electric force. We express $F$ as $F = |q|E$ and solve for $E$:

$$
begin{align*}
|q|E &= ma\
E &= frac{ma}{|q|}
end{align*}
$$

Substituting the known values, we find

$$
begin{align*}
E &= frac{(0.017;mathrm{kg})(3.3;mathrm{m/s}^2)}{5.6times10^{-6};mathrm{C}}\
&= 1.0times10^4;mathrm{N/C}
end{align*}
$$

Result
2 of 2
$10^4$ N/C
Exercise 63
Step 1
1 of 2
$textbf{(a)}$ Positive charges produce an outward-pointing electric field lines. Therefore, the electric field produced by a charge of 5.7 $mu$C points away from the charge.

$textbf{(b)}$ The magnitude of the electric field can be calculated using $E=kq/r^2$, with $r = 0.45$ m:

$$
begin{align*}
E &= kfrac{q}{r^2}\
&= (8.99times10^9;mathrm{N}cdotmathrm{m^2}/mathrm{C^2})timesfrac{5.7times10^{-6};mathrm{C}}{(0.45;mathrm{m})^2}\
&= 2.5times10^5;mathrm{N/C}
end{align*}
$$

Result
2 of 2
$textbf{(a)}$ away                         $textbf{(b)}$ $2.5times10^5$ N/C
Exercise 64
Step 1
1 of 2
$textbf{(a)}$ The field lines emerge from charges $q_1$ and $q_3$ and end at $q_2$. As a result, $q_1$ and $q_3$ must have a positive sign.

$textbf{(b)}$ We see from Figure 20.26 that all the field lines starting from the positive charges $q_1$ and $q_3$ end at the negative charge $q_2$. It follows that the magnitudes of the three charges are related by $|q_1| + |q_3| = |q_2|$. By symmetry (the situation in Figure 20.26 is symmetric about a vertical line passes through $q_2$), $q_1$ and $q_3$ must have the same magnitude. Accordingly, we have

$$
begin{align*}
2|q_1| &= |q_2|\
|q_1| &= frac{|q_2|}{2} = frac{10.0;mumathrm{C}}{2} = 5.00;mumathrm{C}
end{align*}
$$

That we have $q_1 = +5.00;mu$C.

$textbf{(c)}$ It also follows, by symmetry, that $q_3 = q_1 = +5.00;mu$C.

Result
2 of 2
$textbf{(a)}$ positive                        $textbf{(b)}$, $textbf{(c)}$ $+5.00;mu$C
Exercise 65
Step 1
1 of 2
Figure 1 shows a qualitative sketch for the electric field lines produced by two charges, $+q$ and $-q$, separated by a distance $d$. This system of charges is known as an electric dipole. We can see because the charges have equal magnitudes, all the field lines emerging from $+q$ end at $-q$.

Exercise scan

Result
2 of 2
the field line emerge from $+q$ and end at $-q${}
Exercise 66
Step 1
1 of 2
Figure 1 shows a qualitative sketch of the electric field lines produced by two charges, $+q$ and $+2q$, separated by a distance $d$. Note that the field lines due to the individual charges repel each other and extend to infinity. Note also that the number of field lines due to $+2q$ are greater than those due to $+q$ in proportion to the magnitudes of the two charges.

Exercise scan

Result
2 of 2
The outward-emerging field lines due to the individual charges, having densities in proportion to their magnitudes, repel each other and extend to infinity.{}
Exercise 67
Step 1
1 of 2
The situation is sketched below, with each charge at its appropriate position on the corners of an equilateral triangle. Note that the field lines emerge from the positive charges and terminate at the negative charge. We also see the field lines due to the positive charges repel each other and extend to infinity.{}Exercise scan
Result
2 of 2
The field lines start from the positive charges and end at the negative charge. The field lines produced by the positive charges repel each other and extend to infinity.{}
Exercise 68
Step 1
1 of 2
$text{color{#4257b2}Picture the Problem}$

The situation is shown in our sketch, with each charge and the point $P$ (of interest) at its appropriate positions. We also show the electric field produced by each charge. Notice that at $P$ the electric fields due to the individual charges point in the positive $x$ direction; so we expect the total electric field to point in the positive $x$ direction.

$text{color{#4257b2}Strategy}$

The total electric field at $P$ is the vector sum of the electric fields produced by $q_1$ and $q_2$. In particular, notice that $vec{pmb E}_1$ and $vec{pmb E}_2$ add in the same direction. The magnitude of $vec{pmb E}_1$ is $k|q_1|/r^2$, with $r = 4.0$ cm. Similarly, the magnitude of $vec{pmb E}_2$ is $k|q_2|/r^2$, with $r = 8.0$ cm.

$text{color{#4257b2}Solution}$

The electric field produced by $q_1$ at $P$ is

$$
begin{align*}
vec{pmb E}_1 &= kfrac{q_1}{r^2}\
&= (8.99times10^9;mathrm{N}cdotmathrm{m}^2/mathrm{C}^2)frac{(6.2times10^{-6};mathrm{C})}{(4.0times10^{-2};mathrm{m})^2}\
&= 3.5times10^7;mathrm{N/C}qquad(text{positive};x;text{direction})
end{align*}
$$

The electric field produced by $q_2$ at $P$ is

$$
begin{align*}
vec{pmb E}_2 &= kfrac{q_2}{r^2}\
&= (8.99times10^9;mathrm{N}cdotmathrm{m}^2/mathrm{C}^2)frac{(9.5times10^{-6};mathrm{C})}{(8.0times10^{-2};mathrm{m})^2}\
&= 1.3times10^7;mathrm{N/C}qquad(text{positive};x;text{direction})
end{align*}
$$

We then add these two electric fields to find the total electric field, $vec{pmb E}_{rm tot}$, at $P$:

$$
begin{align*}
vec{pmb E}_{rm tot} &= vec{pmb E}_1 + vec{pmb E}_2\
&= 3.5times10^7;mathrm{N/C} + 1.3times10^7;mathrm{N/C}\
&= 4.8times10^7;mathrm{N/C}qquad(text{positive};x;text{direction})
end{align*}
$$

$text{color{#4257b2}Insight}$

The total electric field at $P$ has a magnitude of $4.8times10^7$ N/C, and it points in the positive $x$ direction. Notice that the electric field produced by $q_1$ is greater than that produced by $q_2$, even though $q_2$ has the greater magnitude. The reason is that $q_1$ is three times closer to $P$ than $q_2$ is.

Exercise scan

Result
2 of 2
$4.8times10^7$ N/C in the positive $x$ direction
Exercise 69
Step 1
1 of 2
$textbf{(a)}$ The two configurations (1) and (2) are shown in our sketch, with each charge at its appropriate location in each case. At the center of the square, the electric fields produced by the individual charges at the corners of the squares have the same magnitude (which we will call $E$), as all the charges are identical in magnitude and are the same distance $(d/sqrt{2})$ from the center of the square. We see in configuration (1) the electric fields due to the charges located on opposite corners are directed in opposite directions and cancel each other in pairs. In contrast, we see in configuration (2) the electric fields due to the charges positioned on opposite corners point in the same direction and add up in pairs. As a result, the total electric field at the center of the square will have the greater magnitude in case (2).

$textbf{(b)}$ The magnitude of the electric field produced by each charge on the corners of the square at the center of the square is $E = kq/r^2$, with $r = d/sqrt{2}$.

For case (1), we’ve seen how the electric fields due to the individual charges cancel each other in pairs [see Fig. (1)], leading to zero total electric field at the center of the square.

For case (2), the total electric field at the center of the square point downward [see Fig. (2)] with a magnitude of

$$
4Ecos45^circ = 4kfrac{q}{(d/sqrt{2})^2}cdotfrac{1}{sqrt{2}} = 4sqrt{2};frac{kq}{d^2}
$$

Exercise scan

Result
2 of 2
$textbf{(a)}$ (2)                        $textbf{(b)}$ 0; $4sqrt{2};kq/d^2$
Exercise 70
Step 1
1 of 3
$textbf{(a)}$ The situation is shown in our sketch, with each electric field at its appropriate location. The fact the two electric fields point in the same direction along the $x$ axis tells us the source charge (with magnitude $|q|$) is located on the $x$ axis on either side of the two given locations of the electric field. Moreover, the magnitude of the electric field at ($x = 10.0$ cm, 0) is greater than that at ($x = 5.00$ cm, 0), which indicates the charge is positioned on the right side, where $x > 10.0$ cm. Suppose $x$ is the position of the charge on the $x$-axis. The precise location of the charge can be calculated by imposing the values of the electric-field magnitude at the given locations and solving the resulted equations simultaneously for the value of $x$. The magnitude of the electric field at $x = 5.00$ cm is

$$
E_1 = kfrac{|q|}{(x – 5.00;{rm cm})^2}
$$

Similarly, the magnitude of the electric field at $x = 10.0$ cm is

$$
E_2 = kfrac{|q|}{(x – 10.0;{rm cm})^2}
$$

Dividing $E_2$ by $E_1$ and using the known values of $E_1$ and $E_2$, we have

$$
frac{15.0;{rm N/C}}{10.0;{rm N/C}} = frac{(x – 5.00;{rm cm})^2}{(x – 10.0;{rm cm})^2}
$$

We simplify the result and obtain the following quadratic equation in $x$:

$$
x^2 – (40.0;{rm cm})x + 250;{rm cm}^2 = 0
$$

which has the solutions $x = 32.2$ cm and $x = 7.75$ cm. The latter is not accepted, as it doesn’t satisfy $x > 10.0$ cm, so the correct location of the point charge should be $x = 32.2$ cm.

Exercise scan

Step 2
2 of 3
$textbf{(b)}$ From the location of the point charge and the fact the electric field at the two given positions point in the positive $x$ direction, we can infer that the charge has a negative sign. The magnitude of the charge can be found by substituting $x = 32.2$ cm into the expression for $E_1$ and solving for $q$:

$$
begin{align*}
E_1 &= kfrac{|q|}{(32.2;{rm cm} – 5.00;{rm cm})^2}\
|q| &= frac{(27.2;{rm cm})^2E_1}{k}\
|q| &= frac{(27.2times10^{-2};{rm m})^2(10.0;{rm N/C})}{8.99times10^9;mathrm{N}cdotmathrm{m}^2/mathrm{C}^2}\
&= 82.3;mumathrm{C}
end{align*}
$$

We see the point charge is $q = -82.3;mu$C.

Result
3 of 3
$textbf{(a)}$ 32.2 cm                        $textbf{(b)}$ $-82.3;mu$C
Exercise 71
Step 1
1 of 2
The situation is shown in our sketch, with each charge at its appropriate location. Each charge has a well known magnitude $q$ and the side length of the equilateral triangle is denoted by $d$.

$textbf{(a)}$ As shown in our sketch, at the midpoint of (the right) side of the triangle the electric fields produced by charges 1 and 2 have the same magnitude $(E_1 = E_2)$ and opposite directions, leading to a complete cancelation. As a result, the total electric field is only due to the (uncanceled) electric field produced by charge 3. In such case, the magnitude of the electric field due to charge 3 at the required position is $E_3 = kq/r^2$, with $r = dsqrt{3}/2$, and thus the magnitude of the net electric field is

$$
begin{align*}
E_{rm tot} = E_3 &= kfrac{q}{(dsqrt{3}/2)^2}\
&= kfrac{4q}{3d^2}\
&= (8.99times10^9;mathrm{N}cdotmathrm{m}^2/mathrm{C}^2)timesfrac{4times(4.7times10^{-6};mathrm{C})}{3times(0.21;mathrm{m})^2}\
&= 1.3times10^6;mathrm{N/C}
end{align*}
$$

By symmetry, this is the same magnitude of the electric field at the midpoint of any of the three sides of the triangle (rotate the whole sketch $60^circ$ in either direction and the same result will be obtained).

$textbf{(b)}$ Referring to our sketch again, we can see at the center of the triangle the electric fields produced by the individual charges have the same magnitude and, moreover, are arranged in a three-fold symmetric configuration (the three electric fields are oriented such that the mutual angle between any two of them is exactly $60^circ$). The result is a zero net electric field at the center of the triangle (note that the three vectors can be arranged to form a complete triangle, leading to zero vector sum).

In such case, we can see the magnitude of the electric field at the center of the triangle is less than that at the midpoint of a side.

Exercise scan

Result
2 of 2
$textbf{(a)}$ $1.3times10^6$ N/C                        $textbf{(b)}$ less
Exercise 72
Step 1
1 of 1
The electric potential decreases in the direction of the electric field. Thus ,the electric field will be pointing from positive potential to negative potential in between them.
Exercise 73
Step 1
1 of 1
If the electric field is zero in some region of space, the electric potential need not bezero there as well.As an example , inside a hollow charged sphere ,there is potential but no electric field.
Exercise 74
Step 1
1 of 2
$textbf{(a)}$ The situation is shown in our sketch. An electron $-e$ released from rest in a region of space with an electric field $vec{pmb E}$ experiences an electric force of magnitude $F = eE$ in the opposite direction of $vec{pmb E}$. If the charge moves to the left through a distance $d$, the electric force and the displacement are in the same direction. Therefore, the work done by the electric force is positive and equal in magnitude to force times distance:

$$
W = eEd
$$

The equation from Chapter 6 that relates work and potential energy, $Delta PE = -W$, gives a direct connection between the work done to move charges in an electric field and the system’s change in electric potential energy:

$$
Delta PE = -Wqquadtext{or}qquad Delta PE = -eEd
$$

We see the electric potential energy of the system $decreases$ in this case.

$textbf{(b)}$ The $best$ explanation is the choice $textbf{B.}$

As the electron begins to move, its kinetic energy increases. The increase in kinetic energy is equal to the decrease in the electric potential energy of the system.

Exercise scan

Result
2 of 2
$textbf{(a)}$ decreases                        $textbf{(b)}$ $textbf{B.}$
Exercise 75
Step 1
1 of 2
In order to produce zero electric potential at point A, $q_2$ must have a negative charge, with a slightly $greater$ magnitude than $q_1$ (since A is quite further from $q_2$ than from $q_1$). Applying this result, we conclude the electric potential at point B is $negative$, as B is closer to $q_2$ than to $q_1$ and $q_2$ has the greater magnitude.
Result
2 of 2
negative{}
Exercise 76
Step 1
1 of 2
In order to produce zero electric potential at point B, $q_2$ must have a negative charge, with a little less magnitude than $q_1$ (since B is a bit closer to $q_2$ than to $q_1$). Applying this result, we conclude the electric potential at point A is $positive$, as A is closer to $q_1$ than to $q_2$ and $q_1$ has the greater magnitude.
Result
2 of 2
positive{}
Exercise 77
Step 1
1 of 2
We first note that we needn’t be concerned with the distance factor in the all arrangements, as in each case the charges are the same distance from the origin. It then follows that the total electric potential at the origin in each arrangement will depend only on the algebraic sum of the constituent charges.

In such case, the arrangements A and B correspond to zero total charge each. The arrangement C corresponds to a total charge of $-3q$. The final arrangement D corresponds to a total charge of $+q$. Hence, the rank for the four arrangements in order of increasing electric potential at the origin goes like $rm C < A = B < D$.

Result
2 of 2
$$
rm C < A = B < D
$$
Exercise 78
Step 1
1 of 2
$text{color{#4257b2}Strategy}$

The equation $E = -Delta V/d$ gives us a direction connection between the electric field and the change in the electric potential, from which we can determine the magnitude of the electric field. It also tells us the electric potential decreases in the direction of the electric field.

$text{color{#4257b2}Solution}$

Substitute $d = 0.10;mu$m and $Delta V = -0.070$ V in $|E| = |Delta V|/d$:

$$
|E| = frac{|Delta V|}{d} = frac{0.070;mathrm{V}}{0.10times10^{-6};mathrm{m}} = 7.0times10^5;mathrm{V/m}
$$

Since the electric potential inside the living cell is lower than that outside, then the electric field must point from the outside inward and through the cell membrane toward inside of the cell.

Result
2 of 2
$7.0times10^5$ V/m from the outside to inside of the cell
Exercise 79
Step 1
1 of 2
The proton’s final speed can be found using energy conservation. To do so, we first solve the energy conservation equation for the final kinetic energy:

$$
begin{align*}
frac{1}{2}m_pv_2^2 &= frac{1}{2}m_pv_1^2 + PE_1 – PE_2\
&= frac{1}{2}m_pv_1^2 + e(V_1 – V_2)
end{align*}
$$

Next we set $v_1 = 0$, since the particle starts at rest, and solve for $v_2$:

$$
begin{align*}
frac{1}{2}m_pv_2^2 &= e(V_1 – V_2)\
v_2 &= sqrt{frac{2e(V_1 – V_2)}{m_p}}
end{align*}
$$

Realizing that the proton has a charge of magnitude $e = 1.60times10^{-19}$ C and a mass of $m_p = 1.67times10^{-27}$ kg and substituting the other known values, we find

$$
begin{align*}
v_2 &= sqrt{frac{2(1.60times10^{-19};mathrm{C})(275;mathrm{V})}{(1.67times10^{-27};mathrm{kg})}}\
&= 2.30times10^5;mathrm{m/s}
end{align*}
$$

Result
2 of 2
$2.30times10^5$ m/s
Exercise 80
Step 1
1 of 2
$text{color{#4257b2}Strategy}$

As the object begins to move, its kinetic energy increases. The gained kinetic energy $Delta KE$ is equal to the decrease in the electric potential energy $Delta PE$ of the system. The equation $Delta PE = qDelta V$ gives us a direct connection between the change in the electric potential energy and the corresponding potential difference, from which we can determine the value of $Delta V$.

$text{color{#4257b2}Solution}$

The gained kinetic energy is equal to the change in the electric potential energy:

$$
Delta KE = Delta PE = qDelta V
$$

Solve for $Delta V$ and substitute $Delta KE = 0.0014$ J and $q = 3.1;mu$C:

$$
begin{align*}
Delta V &= frac{Delta KE}{q}\
&= frac{(0.0014;mathrm{J})}{(3.1times10^{-6};mathrm{C})}\
&= 450;mathrm{V}
end{align*}
$$

Result
2 of 2
450 V{}
Exercise 81
Step 1
1 of 2
The final speed of the electroncs can be found using energy conservation. To do so, we first solve the energy conservation equation for the final kinetic energy:

$$
begin{align*}
frac{1}{2}m_ev_2^2 &= frac{1}{2}m_ev_1^2 + PE_1 – PE_2\
&= frac{1}{2}m_ev_1^2 + (-e)(V_1 – V_2)
end{align*}
$$

Next we set $v_1 = 0$ (as the electrons are accelerated from rest) and solve for $v_2$:

$$
begin{align*}
frac{1}{2}m_ev_2^2 &= e(V_2 – V_1)\
v_2 &= sqrt{frac{2e(V_2 – V_1)}{m_e}}
end{align*}
$$

We realize the electrons have a charge of magnitude $e = 1.6times10^{-19}$ C and a mass of $m_e = 9.1times10^{-31}$ kg and substitute the other know values:

$$
begin{align*}
v_2 = &= sqrt{frac{2(1.6times10^{-19};mathrm{C})(25times10^3;mathrm{V})}{9.1times10^{-31};mathrm{kg}}}\
&= 9.4times10^7;mathrm{m/s}
end{align*}
$$

Result
2 of 2
$9.4times10^7$ m/s
Exercise 82
Step 1
1 of 3
The electric potential for a point charge is given by
$$
begin{aligned}
V = k frac{q}{r}
end{aligned}
$$
where $k = 8.99times 10^{9};text{N}cdot text{m}^2/text{C}^2$ is the Coulomb constant, $q$ is the charge, and $r$ is the distance from the charge.

**Given**

Charge of a proton: $e = 1.6times 10^{-19};text{C}$
Radial distance between the proton and the electron’s orbit: $r = 0.529times 10^{-10};text{m}$

Step 2
2 of 3
We substitute the given into the electric potential equation.
$$
begin{aligned}
V &= (8.99times 10^{9};text{N}cdot text{m}^2/text{C}^2) left(frac{1.6times 10^{-19};text{C}}{0.529times 10^{-10};text{m}}right) \ &= boxed{27.2;text{V}}
end{aligned}
$$
Result
3 of 3
$V = 27.2;text{V}$
Exercise 83
Step 1
1 of 2
$text{color{#4257b2}Strategy}$

$textbf{(a)}$ The equation $E = -Delta V/d$ gives us a direct connection between the electric field and the corresponding electric potential, which can be used to solve for the value of $Delta V$. $textbf{(b)}$ It also tells us that if the separation $d$ is increased, the required potential difference $Delta V$ (to obtain the same $E$) increases.

$text{color{#4257b2}Solution}$

$textbf{(a)}$ Substitute $E = 3.0times10^6$ V/m and $d = 0.0635$ cm into $Delta V = -Ed$:

$$
Delta V = -(3.0times10^6;mathrm{V/m})(0.0635times10^{-2};mathrm{m}) = -1.9;mathrm{kV}
$$

$textbf{(b)}$ From the relationship $|Delta V| = Ed$, we can infer that the potential difference, required for the same value of $E$, increases as the separation $d$ of the electrodes is increased.

Result
2 of 2
$textbf{(a)}$ 1.9 kV                         $textbf{(b)}$ increases
Exercise 84
Step 1
1 of 4
A battery is a capacitor as both store energy. The total energy stored in a battery is calculated as
$$
begin{aligned}
E = frac{1}{2}QV
end{aligned}
$$
where $Q$ is the battery’s charge and $V$ is the voltage.

The kinetic energy is calculated using the formula:
$$
begin{aligned}
text{KE} = frac{1}{2}mv^2
end{aligned}
$$

**GIVEN**

Voltage: $V = 12;text{V}$
Charge: $Q = 7.5times 10^{5};text{C}$
Mass: $m = 1400;text{kg}$

Step 2
2 of 4
We solve first for the total energy that can be stored in the battery.
$$
begin{aligned}
E = frac{1}{2}(7.5times 10^{5};text{C})(12;text{V}) = 4.5times 10^{6};text{J}
end{aligned}
$$

Step 3
3 of 4
We isolate $v$ from the kinetic energy formula on one side of the equation. Then we set as $text{KE} = 4.5times 10^{6};text{J}$.
$$
begin{aligned}
v &= sqrt{frac{2text{KE}}{m}} \ &= sqrt{frac{2(4.5times 10^{6};text{J})}{1400;text{kg}}} \ &= boxed{80;text{m/s}}
end{aligned}
$$
Result
4 of 4
$v = 80;text{m/s}$
Exercise 85
Step 1
1 of 2
$text{color{#4257b2}Strategy}$

As the proton slows down its speed to rest, its kinetic energy decreases. The lost kinetic energy $Delta KE$ is equal to the increase in the electric potential energy $Delta PE$ of the system. The kinetic energy of the proton is equal to $m_pv^2/2$ ($m_p$ is the proton’s mass) and the equation $Delta PE = qDelta V$ (with $q = e$ for the proton) gives us a direct connection between the electric potential energy and the corresponding potential difference, from which we can find the value of $Delta V$.

$text{color{#4257b2}Solution}$

The conversion of energy gives us:

$$
Delta KE = Delta PE
$$

which leads to

$$
frac{1}{2}m_pv^2 = eDelta V
$$

Solve for $Delta V$ and substitute the known values ($e = 1.60times10^{-19}$ C and $m_p = 1.67times10^{-27}$ kg for the proton):

$$
begin{align*}
Delta V &= frac{m_pv^2}{2e}\
&= frac{(1.67times10^{-27};mathrm{kg})(4.0times10^5;mathrm{m/s})^2}{2(1.6times10^{-19};mathrm{C})}\
&= 840;mathrm{V}
end{align*}
$$

Result
2 of 2
840 V{}
Exercise 86
Step 1
1 of 4
a) According to Chapter 20.2, the electric potential does not change when the direction is perpendicular to the electric field. Since the direction of the electric potential of points A to B are perpendicular to the electric field, $V_text{A}=V_text{B}$. Therefore,
$$
begin{aligned}
Delta V= V_text{B}-V_text{A}=boxed{0}
end{aligned}
$$
Step 2
2 of 4
b) The electric potential decreases in the direction of the electric field. Since the electric potential from point B to C follows the direction of the electric field, the electric potential decreases. Therefore, **the electric potential at point B is greater than the electric potential at point C.**
Step 3
3 of 4
c) The change in electric potential is given by:
$$
begin{aligned}
Delta V =-Ed
end{aligned}
$$
where $E$ is the magnitude of the electric field and $d$ is the distance.
Step 4
4 of 4
Given the distance from point B to C, 4.0 cm and $E=1200 ;text{N/text{C}}$, we can find the change in electric potential.
$$
begin{aligned}
Delta V =-(1200frac{text{N}}{text{C}})(0.04;text{m})=-48frac{text{N}cdottext{m}}{text{C}}=boxed{-48;text{V}}
end{aligned}
$$
Exercise 87
Step 1
1 of 2
$text{color{#4257b2}Strategy}$

The total electric potential due to multiple charges is equal to the algebraic sum of the potentials due to the individual charges. Therefore, the electric potential at point P, in Fig. 20.30, is the algebraic sum of the potentials due to the given three charges on the vertices of the triangle. In particular, the electric potential due to the $+2.75$-$mu$C charge at P is equal to $V_1 = kq/r$, with $q = +2.75;mu$C and $r = 0.625$ m. Similarly, the electric potential due to the $-1.72$-$mu$C charge at P is equal to $V_2 = kq/r$, with $q = -1.72;mu$C and $r = 0.625$ m. Finally, from the geometry of Fig. 20.30 the separation between the leftover charge (of $+7.45;mu$C) and P is equal to $(1.25;mathrm{m})sqrt{3}/2 = 1.08$ m, so the electric potential due to the $+7.45$-$mu$C charge at P is equal to $V_3 = kq/r$, with $q = +7.45;mu$C and $r = 1.08$ m.

$text{color{#4257b2}Solution}$

The electric potential due to the $+2.75$-$mu$C charge at P is

$$
begin{align*}
V_1 &= (8.99times10^9;mathrm{N}cdotmathrm{m}^2/mathrm{C}^2)timesfrac{(+2.75times10^{-6};mathrm{C})}{(0.625;mathrm{m})}\
&= 4.0times10^4;mathrm{V}
end{align*}
$$

The electric potential due to the $+2.75$-$mu$C charge at P is

$$
begin{align*}
V_2 &= (8.99times10^9;mathrm{N}cdotmathrm{m}^2/mathrm{C}^2)timesfrac{(-1.72times10^{-6};mathrm{C})}{(0.625;mathrm{m})}\
&= -2.5times10^4;mathrm{V}
end{align*}
$$

The electric potential due to the $+7.45$-$mu$C charge at P is

$$
begin{align*}
V_3 &= (8.99times10^9;mathrm{N}cdotmathrm{m}^2/mathrm{C}^2)timesfrac{(+7.45times10^{-6};mathrm{C})}{(1.08;mathrm{m})}\
&= 6.20times10^4;mathrm{V}
end{align*}
$$

Hence, the electric potential at P is

$$
begin{align*}
V &= V_1 + V_2 + V_3\
&= (4.0times10^4;mathrm{V}) + (-2.5times10^4;mathrm{V}) + (6.20times10^4;mathrm{V})\
&= 7.7times10^4;mathrm{V}
end{align*}
$$

Result
2 of 2
$7.7times10^4$ V
Exercise 88
Step 1
1 of 2
$text{color{#4257b2}Strategy}$

The total electric potential due to multiple charges is equal to the algebraic sum of the potentials due to the individual charges. Let us call the point at the empty corner of the rectangle P. The electric potential at point P (see Fig. 20.31) is the algebraic sum of the potentials due to the given three charges on the corners of the rectangle. In particular, the electric potential at P due to the $-3.3$-$mu$C charge is equal to $V_1 = kq/r$, with $q = -3.3;mu$C and $r = 0.25$ m. From the geometry of Fig. 20.31 the separation between the upper-left charge (of $-6.1;mu$C) and P is equal to 0.30 m, so the electric potential at P due to the $-6.1$-$mu$C charge is equal to $V_2 = kq/r$, with $q = -6.1;mu$C and $r = 0.30$ m. Finally, the electric potential at P due to the $+2.7$-$mu$C charge is equal to $V_3 = kq/r$, with $q = +2.7;mu$C and $r = 0.16$ m.

$text{color{#4257b2}Solution}$

The electric potential at P due to the lower-left charge is

$$
begin{align*}
V_1 &= (8.99times10^9;mathrm{N}cdotmathrm{m}^2/mathrm{C}^2)timesfrac{(-3.3times10^{-6};mathrm{C})}{(0.25;mathrm{m})}\
&= -1.2times10^5;mathrm{V}
end{align*}
$$

The electric potential at P due to the upper-left charge is

$$
begin{align*}
V_2 &= (8.99times10^9;mathrm{N}cdotmathrm{m}^2/mathrm{C}^2)timesfrac{(-6.1times10^{-6};mathrm{C})}{(0.30;mathrm{m})}\
&= -1.8times10^5;mathrm{V}
end{align*}
$$

The electric potential at P due to the upper-right charge is

$$
begin{align*}
V_3 &= (8.99times10^9;mathrm{N}cdotmathrm{m}^2/mathrm{C}^2)timesfrac{(+2.7times10^{-6};mathrm{C})}{(0.16;mathrm{m})}\
&= 1.5times10^5;mathrm{V}
end{align*}
$$

Hence, the electric potential at P is

$$
begin{align*}
V &= V_1 + V_2 + V_3\
&= (-1.2times10^5;mathrm{V}) + (-1.8times10^5;mathrm{V}) + (1.5times10^5;mathrm{V})\
&= -1.5times10^5;mathrm{V}
end{align*}
$$

Result
2 of 2
$-1.5times10^5$ V
Exercise 89
Step 1
1 of 2
$text{color{#4257b2}Strategy}$

$textbf{(a)}$ The equation $E = -Delta V/d$ gives us a direct connection between the electric field and the corresponding electric potential. The geometric interpr-
etation of this relationship is that the magnitude of the electric field is given by how much steep the graph of the electric potential versus the position. That the region within which the $V$-$x$ graph (shown in Fig. 20.32) is steep the most is the one with the electric field of the greatest magnitude. $textbf{(b)}$ Using the $V$-$x$ graph shown in Fig. 20.32, we can divide the change in the electric potential $Delta V$ by the corresponding distance $d = Delta x$, to calculate the magnitude of the electric field in each region.

$text{color{#4257b2}Solution}$

$textbf{(a)}$ By inspection, we see region 4 corresponds to the steepest $V$-$x$ line. In such case, the electric field has the greatest magnitude in region 4.

$textbf{(b)}$ In region 1, we have $Delta V = (6.0 – 8.0)$ V = $-2.0$ V and $d = Delta x = (0.15 – 0)$ m = 0.15 m. Thus the electric field in this region has the magnitude

$$
E = -frac{Delta V}{d} = -frac{(-2.0;mathrm{V})}{(0.15;mathrm{m})} = 13;mathrm{V/m}
$$

In region 2, the value of $V$ is constant with $x$; that is, $Delta V = 0$. In such case, the electric field in this region is identically zero.

In region 3, we have $Delta V = (8.0 – 6.0)$ V = $2.0$ V and $d = Delta x = (0.60 – 0.25)$ m = 0.35 m. Thus the electric field in this region has the magnitude

$$
E = frac{Delta V}{d} = frac{(2.0;mathrm{V})}{(0.35;mathrm{m})} = 5.7;mathrm{V/m}
$$

Finally, in region 4 we have $Delta V = (1.0 – 8.0)$ V = $-7.0$ V and $d = Delta x = (0.70 – 0.60)$ m = 0.10 m. Thus the electric field in this region has the magnitude

$$
E = -frac{Delta V}{d} = -frac{(-7.0;mathrm{V})}{(0.10;mathrm{m})} = 70;mathrm{V/m}
$$

$text{color{#4257b2}Insight}$

As we can see from the results in part (b), the electric field has the greatest magnitude (of 70 V/m) in region 4, which corresponds to the steepest part of the $V$-$x$ graph.

Result
2 of 2
$textbf{(a)}$ 4                        $textbf{(b)}$ 13 V/m, 0, 5.7 V/m, 70 V/m
Exercise 90
Step 1
1 of 2
The capacitance of a capacitor is given by
$$
begin{aligned}
C=frac{Q}{V}
end{aligned}
$$
wherein $Q$ is the magnitude of the charge in either plate and $V$ is the magnitude of the voltage difference between its plates.
Step 2
2 of 2
From the equation, the greater the electric potential difference $V$ for a given charge $Q$, the lesser the capacitance $C$ of the capacitor. Since capacitor 1 has greater voltage between its plates than capacitor 2, **the capacitance of capacitor 1 is less than the capacitance of capacitor 2**.
Exercise 91
Step 1
1 of 1
A capacitor is like a bucket of water. In this water-bucket analogy to the capacitor, the **amount of water in the bucket** is identified as the amount of charge $Q$ stored in the capacitor.
Exercise 92
Step 1
1 of 1
From the water-bucket analogy to capacitors, the potential difference $V$ between plates of the capacitor corresponds to **the depth of the water** in the bucket.
Exercise 93
Step 1
1 of 1
A bucket of water gives an analogy about capacitors. The cross-sectional area of a water bucket, in the analogy, corresponds to the **capacitance $C$ of the capacitor**.
Exercise 94
Step 1
1 of 2
If the area of the plat is increased, the capacitance goes up. This is as you would expect. After all, larger plates have more area, and this gives the plates more room to store charge. It’s just like the analogy with a bucket of water—a capacitor with a large plate area is like a bucket with a large cross-sectional area.{}
Result
2 of 2
increases{}
Exercise 95
Step 1
1 of 2
The dependence of capacitance on plate separation is just the opposite of its dependence on area. If the plate separation is increased, the capacitance decreases. The reason is that a larger separation between the plates increases the potential difference between them. This means that higher voltage is required to store a given amount of charge—which is another way of saying that the capacitance is less, since we have $C = Q/V$.
Result
2 of 2
decreases{}
Exercise 96
Step 1
1 of 3
The capacitance of a capacitor is given by
$$
begin{aligned}
C=frac{Q}{V}
end{aligned}
$$
wherein $Q$ is the magnitude of charge on either plate and $V$ is the electric potential difference.
Step 2
2 of 3
Rearranging this equation, we can get the magnitude of the charge on each capacitor’s plate. Moreover, we substitute the known values, $C=4.1;mu text{F}$ and $V=12;text{V}$, to the rearranged equation.
$$
begin{aligned}
Q&=CV\&=(4.1times10^{-6};text{F})(12;text{V})\&=boxed{49.2;mutext{C}}
end{aligned}
$$
Result
3 of 3
$Q = 49.2;mutext{C}$
Exercise 97
Step 1
1 of 3
The voltage required to give the plates of a capacitor, with capacitance $C$, a charge $Q$ is given by:
$$
begin{aligned}
V=frac{Q}{C}
end{aligned}
$$
Step 2
2 of 3
Having known values of $C=320;text{pF}$ and $Q=8.2times10^{-9};text{C}$, we can solve for the voltage required. Note that pF is equivalent to $10^{-12};text{F}$.
$$
begin{aligned}
V&=frac{8.2times10^{-9};text{C}}{320times10^{-12};text{F}}\&=boxed{25.6;text{V}}
end{aligned}
$$
Result
3 of 3
$V=25.6;text{V}$
Exercise 98
Step 1
1 of 3
The capacitance of a capacitor is given by
$$
begin{aligned}
C=frac{Q}{V}
end{aligned}
$$
wherein $Q$ is the magnitude of charge on either plate and $V$ is the electric potential difference.
Step 2
2 of 3
Given a charge $Q=6.9times10^{-9};text{C}$ and an electric potential difference $V=1.5;text{V}$, we can find the capacitance $C$ through the above equation.
$$
begin{aligned}
C&=frac{6.9times10^{-9};text{C}}{1.5;text{V}}\&=boxed{4.6;text{nF}}
end{aligned}
$$
Result
3 of 3
$C=4.6;text{nF}$
Exercise 99
Step 1
1 of 2
$text{color{#4257b2}Picture the Problem}$

The capacitor is shown in our sketch. Notice that the charge on each plate has the magnitude $Q$. The electric field is uniform and points from the positive plate to the negative plate.

$text{color{#4257b2}Strategy}$

$textbf{(a)}$ Capacitance is defined as $C = Q/V$. We can use this expression to find the capacitance of our capacitor; the charge and the voltage are given in the problem statement. $textbf{(b)}$ In Lesson 20.2 we saw that the electric field is $E = -Delta V/d$. It follows that the magnitude of the electric field is $|E| = |Delta V|/d$. The voltage difference, $Delta V$, is the battery voltage $V$ given in the problem statement. The distance $d$ is the distance between the plates. Therefore, $d = 0.45$ mm.

$text{color{#4257b2}Solution}$

$textbf{(a)}$ Use $C = Q/V$ to find capacitance of the capacitor:

$$
C = frac{Q}{V} = frac{1.1times10^{-8};mathrm{C}}{9.0;mathrm{V}} = 1.2;mathrm{nF}
$$

$textbf{(b)}$ The magnitude of the electric field between the plates is $|E| = |Delta V|/d$, with $d = 0.45$ mm = $0.45times10^{-3}$ m:

$$
|E| = frac{|Delta V|}{d} = frac{9.0;mathrm{V}}{0.45times10^{-3};mathrm{m}} = 2.0times10^4;mathrm{V/m}
$$

Exercise scan

Result
2 of 2
$textbf{(a)}$ 1.2 nF                        $textbf{(b)}$ $2.0times10^4$ V/m
Exercise 100
Step 1
1 of 4
The magnitude of the electric potential $V$ between capacitor plates is given by
$$
begin{aligned}
V=frac{Q}{C}=lvert E rvert d
end{aligned}
$$
where $Q$ is the magnitude of the charge on either plate, $C$ is the capacitance of the capacitor, $lvert E rvert$ is the magnitude of the electric field and $d$ is the plate separation.
Step 2
2 of 4
We can rearrange the equation such that the capacitance $C$ is isolated to one side.
$$
begin{aligned}
C=frac{Q}{lvert E rvert d}
end{aligned}
$$
Step 3
3 of 4
Substituting the known values,$Q=6.7times10^{-9};text{C}$, $lvert E rvert=29000frac{text{V}}{text{m}}$ and $d=0.35times10^{-3};text{m}$, we get the capacitance $C$.
$$
begin{aligned}
C&=frac{6.7times10^{-9};text{C}}{29000frac{text{V}}{text{m}} (0.35times10^{-3};text{m})}\&=boxed{660.1;text{pF}}
end{aligned}
$$
Result
4 of 4
$C=660.1;text{pF}$
Exercise 101
Step 1
1 of 2
$text{color{#4257b2}Strategy}$

$textbf{(a)}$ The energy stored in the capacitor is given by $PE = frac{1}{2}QV$. Using the definition of the capacitance $C = Q/V$, we have $Q = CV$ and so $PE = frac{1}{2}CV^2$. As the motor lifts the mass, the energy in the capacitor is converted to a gravitational potential energy of magnitude $mtextit{g}h$, with $m = 5.0$ g and $h$ being the height to which the mass is lifted. We can solve the equation $frac{1}{2}CV^2 = mtextit{g}h$ for $h$ to find its value. $textbf{(b)}$ This time we apply the procedure in part (a) backwards. That we use the expression for the potential energy $mtextit{g}h$, with $h = 1.0$ cm, to calculate the energy required to lift the mass by the motor to that given height. We then use the energy-conversion equation $frac{1}{2}CV^2 = mtextit{g}h$ to solve for the capacitor’s voltage $V$ (the target variable).

$text{color{#4257b2}Solution}$

$textbf{(a)}$ Solve the equation $frac{1}{2}CV^2 = mtextit{g}h$ for $h$:

$$
h = frac{CV^2}{2mtextit{g}}
$$

Substitute $C = 0.22;mu$F = $0.22times10^{-6}$ F, $V = 1.5$ V, and $m = 5.0$ g = $5.0times10^{-3}$ kg, to find $h$:

$$
h = frac{(0.22times10^{-6};mathrm{F})(1.5;mathrm{V})^2}{2(5.0times10^{-3};mathrm{kg})(9.8;mathrm{m/s}^2)} = 5.1;mumathrm{m}
$$

$textbf{(b)}$ The energy required to lift the mass to a height of $h = 1.0$ cm is

$$
mtextit{g}h = (5.0times10^{-3};mathrm{kg})(9.8;mathrm{m/s}^2)(1.0times10^{-2};mathrm{m}) = 490;mumathrm{J}
$$

Use $frac{1}{2}CV^2 = mtextit{g}h$ to solve for $V$:

$$
begin{align*}
frac{1}{2}CV^2 &= 490;mumathrm{J}\
V &= sqrt{frac{2(490;mumathrm{J})}{C}}
end{align*}
$$

Substitute $C = 0.22;mu$F to find $V$:

$$
V = sqrt{frac{2(490;mumathrm{J})}{0.22;mumathrm{F}}} = 67;mathrm{V}
$$

Result
2 of 2
$textbf{(a)}$ 5.1 $mu$m                         $textbf{(b)}$ 67 V
Exercise 102
Step 1
1 of 7
a) The equation for the magnitude of the charge $Q$ on each plate of a capacitor is given by
$$
begin{aligned}
Q=VC
end{aligned}
$$
where $V$ is the electric potential difference and $C$ is the capacitance of the capacitor.
Step 2
2 of 7
Substituting our known values, $V=550;text{V}$ and $C=430;text{pF}$, to the equation, the charge $Q$ would be:
$$
begin{aligned}
Q&=(550;text{V})(430times10^{-12};text{F})\&=boxed{236.5;text{nC}}
end{aligned}
$$
Step 3
3 of 7
b) The total energy $PE$ stored in a capacitor with charge $Q$ and potential difference $V$ is given by:
$$
begin{aligned}
PE=frac{1}{2}QV
end{aligned}
$$
Step 4
4 of 7
We again substitute our known values to the above equation.
$$
begin{aligned}
PE&=frac{1}{2}(236.5times10^{-9};text{C})(550;text{V})\&=boxed{65.04;mutext{J}}
end{aligned}
$$
Step 5
5 of 7
c) The magnitude of the electric field $lvert E rvert$ between plates is given by:
$$
begin{aligned}
lvert E rvert= frac{lvert Delta V rvert}{d}
end{aligned}
$$
where $d$ is the separation distance between capacitor plates. The plates are separated by a distance of 0.89 mm.
Step 6
6 of 7
Using this value and the potential difference $V$, we would obtain the magnitude of the electric field.
$$
begin{aligned}
lvert E rvert&= frac{550;text{V}}{0.89times10^{-3};text{m}}\ &=boxed{6.18times10^{5}frac{V}{m}}
end{aligned}
$$
Result
7 of 7
a) $Q=236.5;text{nC}$
b) $PE=65.04;mutext{J}$
c) $lvert E rvert=6.18times10^{5}frac{V}{m}$
Exercise 103
Step 1
1 of 2
We have shown in Problem 95 how the capacitance of a capacitor decreases when the spacing between its plates is increased. Furthermore, we know that for a fixed voltage difference $V$ across the capacitor the amount of charge $Q$ on the capacitor’s plates is directly proportional to its capacitance $C$, since we have $Q = CV$. As a result, the magnitude of the charge on the plates also decreases, as the plate separation is increased.
Result
2 of 2
decreases{}
Exercise 104
Step 1
1 of 2
$textbf{(a)}$ The situation is shown in our sketch. A proton (of charge $e$) released from rest in a region of space with a nonzero electric field $vec{pmb E}$ experiences an electric force of magnitude $F = eE$ in the direction of $vec{pmb E}$. If the charge moves to the right through a distance $d$, the electric force and the displacement are in the same direction. Therefore, the work done by the electric force is positive and equal in magnitude to force times distance:

$$
W = eEd
$$

The equation from Chapter 6 that relates work and potential energy, $Delta PE = -W$, gives a direct connection between the work done to move charges in an electric field and the system’s change in electric potential energy:

$$
Delta PE = -Wqquadtext{or}qquad Delta PE = -eEd
$$

We see the electric potential energy of the system $decreases$ in this case.

$textbf{(b)}$ The $best$ explanation is the choice $textbf{A.}$

As the proton begins to move, its kinetic energy increases. The increase in kinetic energy is equal to the decrease in the electric potential energy of the system.

Exercise scan

Result
2 of 2
$textbf{(a)}$ decreases                        $textbf{(b)}$ $textbf{A.}$
Exercise 105
Step 1
1 of 2
$textbf{(a)}$ The electric field between the plates is uniform. Therefore, the electric field between the plates $remains;the;same$ as the the separation of the plates is increased.

$textbf{(b)}$ The magnitude of the electric field is related to the potential difference between the plates by the relationship $|E| = V/d$, or $V = |E|d$. Since $|E|$ remains unchanged between the plates, then the potential difference $V$ between the plates $increases$ as the separation $d$ of the plates is increased.

$textbf{(c)}$ The capacitance is defined as $C = Q/V$. Since $Q$ is constant, an increase in the separation of the plates will lead to an increase in the potential difference $V$, which in turn leads to a $decrease$ in the capacitance $C$.

$textbf{(d)}$ The energy stored in the capacitor is $PE = frac{1}{2}QV$. As the separation of the plates is increased, the voltage difference between the plates $V$ increases, and thus the energy $PE$ stored in the capacitor also $increases$.

Result
2 of 2
$textbf{(a)}$ remains the same                $textbf{(b)}$ increases                $textbf{(c)}$ decreases                $textbf{(d)}$ increases
Exercise 106
Step 1
1 of 2
$textbf{(a)}$ The electric field is related to the potential difference between the plates by the relationship $E = -V/d$. Therefore, if the spacing between the plates is increased, the electric field between the plates $decreases$ in magnitude.

$textbf{(b)}$ If the plate separation is increased, the charge on the plates $decreases$. This is what you would expect when the electric field between the plates is reduced.

$textbf{(c)}$ The capacitance is defined as $C = Q/V$. The potential difference $V$ between the plates is constant. Therefore, an increase in the spacing between the plates will lead to a decrease in the charge $Q$ on the plates, which ultimately leads to a $decrease$ in the capacitance $C$.

$textbf{(d)}$ The energy stored in the capacitor is $PE = frac{1}{2}QV$. For a fixed potential difference $V$, the decrease in the plate charge $Q$, when the plate separation is increased, $deceases$ the energy $PE$ stored in the capacitor.

Result
2 of 2
$textbf{(a)}$ decreases                $textbf{(b)}$ decreases                $textbf{(c)}$ decreases                $textbf{(d)}$ decreases
Exercise 107
Step 1
1 of 2
According to Chapter 20.1, an electric field’s direction is away from a positive charge and goes toward a negative charge, such that the field lines start from the positive charge and end in the negative charge.
Step 2
2 of 2
Since the electric catfish’s muscle near the head produces positive charges and the region near the tail produces negative charges, **the direction of the electric field produced by the catfish is from the region near the head and goes towards the region near the tail.**
Exercise 108
Step 1
1 of 2
The capacitance of a parallel plate capacitor is inversely proportional to the distance between the plates so we increase the capacitance by depressing the key.
Result
2 of 2
Click here for the solution.
Exercise 109
Step 1
1 of 2
The change in proton’s kinetic energy equals the work done by the electric force so we can write

$$
KE_2-KE_1 = frac{1}{2}mv^2 – 0 = e(V_2-V1) = eEd.
$$

Now we get

$$
v=sqrt{2frac{eEd}{m}} =sqrt{2frac{1.6times10^{-19}text{ C}1.08times10^{8}text{ N/C}times 0.5text{ m}}{1.67times 10^{-27}text{ kg}}} = 3.22times 10^{6}text{ m/s}.
$$

Result
2 of 2
Click here for the solution.
Exercise 110
Step 1
1 of 4
The change in electric potential $Delta V$ is given by the equation:
$$
begin{aligned}
Delta V = frac{Delta PE}{q}
end{aligned}
$$
such that $Delta PE$ is the change in electrical potential energy and $q$ is the amount of charge.
Step 2
2 of 4
Since we are given the work done by the electric field, 0.052 J, we need the equation be in terms of work $W$. Note that the relationship of the change in electric potential energy $Delta PE$ and work $W$ is given by:
$$
begin{aligned}
Delta PE =-W
end{aligned}
$$
Step 3
3 of 4
Substituting this expression and known values to the equation for $Delta V$, we get:
$$
begin{aligned}
Delta V &= frac{-W}{q}\&=frac{-0.052;text{J}}{+5.7times10^{-6};text{C}}\&=boxed{-9.1;text{kV}}
end{aligned}
$$
Result
4 of 4
$Delta V= -9.1;text{kV}$
Exercise 111
Step 1
1 of 2
$text{color{#4257b2}Picture the Problem}$

The situation is shown in our sketch, along with the free-body diagram for the thread. Notice that we choose the upward direction as the positive $y$ direction.

$text{color{#4257b2}Strategy}$

$textbf{(a)}$ The charges on the two given objects have the same sign (positive), so the electric field at the position of the upper charge (of magnitude $q_0$) due to the lower charge (of magnitude $q$) is directed upward. Moreover, it has a magnitude equal to $kq/d^2$, with $d = 1.2$ m. $textbf{(b)}$ There are three forces acting on the upper charge; its downward weight $mtextit{g}$, the upward tension $T$ in the thread, and the upward electric force $F$ exerted by the lower charge $q$. Notice that the magnitude of the electric force $F$ is equal to $q_0E$, where $E$ is the magnitude of the electric field at the position of $q_0$ due to $q$ (given by part (a)). We apply Newton’s first law to the upper charge along the vertical direction ($y$ direction) to obtain an expression for $T$ in terms of $F$ and $mtextit{g}$, from which we can find the value of $T$.

$text{color{#4257b2}Solution}$

$textbf{(a)}$ The electric field at $q_0$ due to $q$ is

$$
begin{align*}
E &= kfrac{q}{d^2}\
&= (8.99times10^9;mathrm{N}cdotmathrm{m}^2/mathrm{C}^2)timesfrac{4.2times10^{-6};mathrm{C}}{(1.2;mathrm{m})^2}\
&= 2.6times10^4;mathrm{N/C}qquad text{(upward)}
end{align*}
$$

$textbf{(b)}$ We apply Newton’s first law to the upper charge in the $y$ direction:

$$
Sigma F_y = F + T – mtextit{g} = 0
$$

We solve the equation for $T$ and express $F$ as $q_0E$:

$$
begin{align*}
T &= mtextit{g} – F\
T &= mtextit{g} – q_0E
end{align*}
$$

We substitute $m = 0.0150$ kg, $q_0 = 3.1;mu$C = $3.1times10^{-6}$ C, and $E = 2.6times10^4$ N/C (from part (a)), and find $T$:

$$
begin{align*}
T &= (0.0150;mathrm{kg})(9.8;mathrm{m/s}^2) – (3.1times10^{-6};mathrm{C})(2.6times10^4;mathrm{N/C})\
&= 0.066 ;mathrm{N}
end{align*}
$$

Exercise scan

Result
2 of 2
$textbf{(a)}$ $2.6times10^4$ N/C (upward)                        $textbf{(b)}$ 0.066 N
Exercise 112
Step 1
1 of 3
$text{color{#4257b2}Strategy}$

$textbf{(a)}$ The electric potential at any of the given points is equal to the electric potential due to the positive charge in positive sign plus the electric potential due the negative charge in negative sign. As a result, the electric potential is smallest in value at the closest point to the point halfway between the two charges. $textbf{(b)}$ The given two charges have the same magnitude, but the positive charge is closer to the given points (ruling out the point A) in each case than the negative charge is. This means the total electric potential at each of the given points is positive (the electric potential is zero at A). Since the magnitude of the electric potential at any point depends inversely on the distance, it follows that the electric potential has the greatest (positive) value at that point as close as possible to the positive charge and at the same time as further as possible from the negative charge. $textbf{(c)}$ At each point of interest, the electric potential is the algebraic sum of the electric potentials due to the two charges. In particular, the electric potential due to either charge $q$ at a point $r$ distance from such charge is equal to $kq/r$.

$text{color{#4257b2}Solution}$

$textbf{(a)}$ The point A (located at the origin) is exactly halfway between the two opposite-in-sign charges. Hence, the electric potential is smallest in value (numerically zero) at A.

$textbf{(b)}$ The remaining points, B, C, and D, are all the same distance from the positive charge. However, from the geometry of Fig. 20.34 the points B and D are the same distance from the negative charge, which is $smaller$ in length than that of point C from the negative charge (this comes as a consequence of the triangle inequality). Hence, the electric potential due to the negative charge at C is the least negative, which when combined with the positive potential gives an ultimate electric potential at C as the most positive (having the greatest value).

Step 2
2 of 3
$textbf{(c)}$ At point A, the electric potential is identically zero.

At point C, the electric potential due to the positive charge is $V_+ = +k|q|/r_+$, with $|q| = 1.2;mu$C and $r_+ = 0.50$ m and the electric potential due to the negative charge is $V_- = -k|q|/r_-$, with $|q| = 1.2;mu$C and $r_- = 1.5$ m. In such case, the electric potential at C is the sum $V_{rm C} = V_+ + V_-$:

$$
begin{align*}
V_{rm C} &= +kfrac{|q|}{r_+} – kfrac{|q|}{r_-}\
&= k|q|left(frac{1}{r_+} – frac{1}{r_-}right)\
&= (8.99times10^9;mathrm{N}cdotmathrm{m}^2/mathrm{C}^2)(1.2times10^{-6};mathrm{C})\
&timesleft(frac{1}{0.50;mathrm{m}} – frac{1}{1.5;mathrm{m}}right)\
&= 14;mathrm{kV}
end{align*}
$$

The points B and D are the same distance from each of the two charges. Hence, the total electric potential at either point is the same and equal to the positive electric potential $V_+ = +k|q|/r_+$, with $|q| = 1.2;mu$C and $r_+ = 0.50$ m, plus the negative electric potential $V_- = -k|q|/r_-$, with $|q| = 1.2;mu$C and $r_- = sqrt{(1.0;mathrm{m})^2 + (0.50;mathrm{m})^2} = 1.1$ m. That the electric potential at B or D is

$$
begin{align*}
V_{rm B} = V_{rm D} &= +kfrac{|q|}{r_+} – kfrac{|q|}{r_-}\
&= k|q|left(frac{1}{r_+} – frac{1}{r_-}right)\
&= (8.99times10^9;mathrm{N}cdotmathrm{m}^2/mathrm{C}^2)(1.2times10^{-6};mathrm{C})\
&timesleft(frac{1}{0.50;mathrm{m}} – frac{1}{1.1;mathrm{m}}right)\
&= 12;mathrm{kV}
end{align*}
$$

$text{color{#4257b2}Insight}$

As we see from the results of part (c), the electric potential does have its smallest value at point A and its greatest value at point C.

Result
3 of 3
$textbf{(a)}$ A

$textbf{(b)}$ C

$textbf{(c)}$ 0, 12 kV, 14 kV, 12 kV

Exercise 113
Step 1
1 of 4
$text{color{#4257b2}Strategy}$

$textbf{(a)}$ The electric potential at any of the given points are the sum of the positive-signed values of the electric potentials due to the individual positive charges. As a result, the electric potential is smallest in value at the point located as further as possible from the location of each charge. $textbf{(b)}$ In contrast to part (a), the electric potential has its greatest value at the point the closest possible to the location of each charge. $textbf{(c)}$ At each point of interest, the electric potential is the sum of the electric potentials due to each individual charge. In particular, the electric potential due to either charge $q$ at a point distance $r$ from the location of $q$ is equal to $kq/r$.

$text{color{#4257b2}Solution}$

$textbf{(a)}$ Referring to the geometry of Fig. 20.34, the point C is the farthest possible from the location of each charge (again the triangle inequality comes into the scene). Hence, the electric potential is smallest in value at C.

$textbf{(b)}$ The point A is the closest possible to the location of each charge. Hence, the electric potential must have its greatest value at A.

Step 2
2 of 4
$textbf{(c)}$ The two (positive) charges have the same magnitude and are the same distance from point A. It follows that the electric potential due to each charge have the same value at A, which is equal to $+kq/r$, with $q = 1.2;mu$C and $r = 0.50$ m. In such case, the total potential at A is the double of this value:

$$
begin{align*}
V_{rm A} &= 2kfrac{q}{r}\
&= 2(8.99times10^9;mathrm{N}cdotmathrm{m}^2/mathrm{C}^2)timesfrac{1.2times10^{-6};mathrm{C}}{0.50;mathrm{m}}\
&= 43;mathrm{kV}
end{align*}
$$

At point C, the electric potential is the sum of the electric potential due to the left charge, $V_{ell} = +kq/r_{ell}$, with $q = 1.2;mu$C and $r_ell = 1.5$ m, and the electric potential due to the right charge, $V_{rm r} = +kq/r_{rm r}$, with $q = 1.2;mu$C and $r_{rm r} = 0.50$ m:

$$
begin{align*}
V_{rm C} &= kfrac{q}{r_ell} + kfrac{q}{r_{rm r}}\
&= kqleft(frac{1}{r_ell} + frac{1}{r_{rm r}}right)\
&= (8.99times10^9;mathrm{N}cdotmathrm{m}^2/mathrm{C}^2)(1.2times10^{-6};mathrm{C})times\
&left(frac{1}{1.5;mathrm{m}} + frac{1}{0.50;mathrm{m}}right)\
&= 29;mathrm{kV}
end{align*}
$$

The points B and D are the same distance from the location of each charge. Hence, the total electric potential at either point has the same value, which is equal to the electric potential due to the left charge, $V_{ell} = +kq/r_{ell}$, with $q = 1.2;mu$C and $r_ell = sqrt{(1.0;mathrm{m})^2 + (0.50;mathrm{m})^2} = 1.1$ m, plus the electric potential due to the right charge, $V_{rm r} = +kq/r_{rm r}$, with $q = 1.2;mu$C and $r_{rm r} = 0.50$ m:

$$
begin{align*}
V_{rm B} = V_{rm D} &= kfrac{q}{r_ell} + kfrac{q}{r_{rm r}}\
&= kqleft(frac{1}{r_ell} + frac{1}{r_{rm r}}right)\
&= (8.99times10^9;mathrm{N}cdotmathrm{m}^2/mathrm{C}^2)(1.2times10^{-6};mathrm{C})times\
&left(frac{1}{1.1;mathrm{m}} + frac{1}{0.5;mathrm{m}}right)\
&= 31;mathrm{kV}
end{align*}
$$

Step 3
3 of 4
$text{color{#4257b2}Insight}$

As we see from the results of part (c), the electric potential does have its smallest value at point C and its greatest value at point A.

Result
4 of 4
$textbf{(a)}$ C

$textbf{(b)}$ A

$textbf{(c)}$ 43 kV, 31 kV, 29 kV, 31 kV

Exercise 114
Step 1
1 of 10
The equation for Newton’s second law of motion is expressed as:
$$
begin{aligned}
Sigma F_{text{net}}=ma
end{aligned}
$$
such that $Sigma F_text{net}$ is the summation of the forces acting on the a body.
Step 2
2 of 10
The sum of the forces acting on the ball in the problem is given by:
$$
begin{aligned}
Sigma F_{text{net}}=F_text{g}-F_text{e}=mg-qE
end{aligned}
$$
wherein $F_text{g}$ is the force of gravity acting on the ball and $F_text{e}$ is the opposing electric force.
Step 3
3 of 10
Note that the expression for the electric field is given by:
$$
begin{aligned}
E=-frac{Delta V}{d}
end{aligned}
$$
Step 4
4 of 10
We then substitute this expression to the equation for the summation of forces and rearrange the equation such that $Delta V$ is isolated.
$$
begin{aligned}
Sigma F_{text{net}}&=mg+qfrac{Delta V}{d}=ma \
Delta V&=frac{m(a-g)d}{q} tag{1}
end{aligned}
$$
Step 5
5 of 10
In order to use Equation (1), we first need to obtain the acceleration due to gravity $g$ through the first half of the problem wherein the ball is uncharged. In getting $g$, we can use one kinematic equation:
$$
begin{aligned}
y=y_0+v_0t-frac{1}{2}gt^2
end{aligned}
$$
Step 6
6 of 10
Rearranging the above equation such that $g$ is isolated to one side and substituting the known values in the problem, $v_0=0;text{m/s}$, $y_0 = 1.00;text{m}$, $y=0;text{m}$ and $t=0.552;text{s}$, we get:
$$
begin{aligned}
g&=-frac{2(y-y_0-v_0t)}{t^2} \& =-frac{2(0;text{m}-1.00text{m}-(0;text{m/s})(0.552;text{s}))}{(0.552;text{s})^2} \ &=6.56;text{m}/text{s}^2
end{aligned}
$$
Step 7
7 of 10
We next find the net acceleration $a$ of the ball through the second half of the problem wherein the ball has a net charge of $7.75;mutext{C}$. We can use the expression obtained for $g$ but we replace it with $a$. With the charged ball, the time $t$ has a value of $0.680;text{s}$.
$$
begin{aligned}
a&=-frac{2(y-y_0-v_0t)}{t^2} \ &=-frac{2(0;text{m}-1.00text{m}-(0;text{m/s})(0.680;text{s}))}{(0.680;text{s})^2} \ &=4.33;text{m}/text{s}^2
end{aligned}
$$
Step 8
8 of 10
To find $Delta V$, we use Equation (1) and substitute our obtained and known values in the problem.
$$
begin{aligned}
Delta V&=frac{m(a-g)d}{q}\&=frac{(0.250;text{kg})(4.33;text{m}/text{s}^2-6.56;text{m}/text{s}^2)(1.00;text{m})}{7.75times10^{-6};text{C}}\&=-7.22times10^{4};text{V}
end{aligned}
$$
Step 9
9 of 10
However, we are interested in the initial electric potential $V_text{i}$ at a height $1.00;text{m}$ above the ground. We can find $V_text{i}$ using the potential difference $Delta V$ and the final electric potential $V_text{f}$ which has a value of $0;text{V}$.
$$
begin{aligned}
Delta V&=V_text{f}-V_text{i}\
V_text{i}&=V_text{f}-Delta V\&=0;text{V}-(-7.22times10^{4};text{V})\&=boxed{7.22times10^{4};text{V}}
end{aligned}
$$
Result
10 of 10
$Delta V = 7.22times10^{4};text{V}$
Exercise 115
Step 1
1 of 8
The figure shown in the problem resembles a projectile motion. The position-time equations for projectiles is given by
$$
begin{aligned}
x_text{f}&=x_text{i}+v_text{x,i}t\
y_text{f}&=y_text{i}+v_text{y,i}t-frac{1}{2}a_text{y}t^2
end{aligned}
$$
Step 2
2 of 8
Using Equation (1), we can get the time for which the electron travels the horizontal distance, $2.25times10^{-2};text{m}$. We assume that $x_text{i}=0;text{m}$ and we have a initial speed of $5.45times10^6;text{m/s}$.
$$
begin{aligned}
t=frac{x_text{f}-x_text{i}}{v_text{x,i}}=frac{2.25times10^{-2};text{m}-0;text{m}}{5.45times10^6;text{m/s}}=4.1times10^{-9};text{s}
end{aligned}
$$
Step 3
3 of 8
From Equation (2), we can get an expression for the acceleration $a_text{y}$. Note that $v_text{y,i}=0;text{m/s}$.
$$
begin{aligned}
y_text{f}&=y_text{i}-frac{1}{2}a_text{y}t^2\
a_text{y}&=2frac{y_text{i}-y_text{f}}{t^2}
end{aligned}
$$
Step 4
4 of 8
Another expression for $a_text{y}$ could be obtained through the the summation of forces in the y-direction and through Newton’s second law of motion.
$$
begin{aligned}
Sigma F_text{y}=ma_text{y}\
F_text{e}=ma_text{y}\
a_text{y}=frac{F_text{e}}{m}
end{aligned}
$$
Step 5
5 of 8
The electric force $F_text{e}$ can be rewritten in terms of the magnitude of the electric field $E$. Therefore, $a_text{y}$ would be:
$$
begin{aligned}
a_text{y}=frac{Eq_0}{m}
end{aligned}
$$
Step 6
6 of 8
Equating the two expressions for $a_text{y}$, we can get an expression for the magnitude of the electric field $E$.
$$
begin{aligned}
frac{Eq_0}{m}&=2frac{y_text{i}-y_text{f}}{t^2}\
E&=2frac{m(y_text{i}-y_text{f})}{q_0t^2}
end{aligned}
$$
Step 7
7 of 8
We then substitute our known values, $m_text{e}=9.11times10^{-31};text{kg}$, $q_0=1.60times10^{-19};text{C}$, $y_text{i}=0;text{m}$, $y_text{f}=-0.618times10^{-2};text{m}$ and $t=4.1times10^{-9};text{s}$, we get the magnitude of the electric field $E$.
$$
begin{aligned}
E&=2left(frac{9.11times10^{-31};text{kg}(0;text{m}-(-0.618times10^{-2};text{m}))}{(1.60times10^{-19};text{C})(4.1times10^{-9};text{s})^2}right)\&=boxed{4129;text{N/C}}
end{aligned}
$$
Result
8 of 8
$E=4129;text{N/C}$
Exercise 116
Step 1
1 of 8
Let there be three charges, $q_1$, $q_2$ and $q_3$ such that $q_1=24.5;mutext{C}$, $q_2=-11.2;mutext{C}$ and $q_3$ is the unknown charge. For each point charge, we first need to find the distance $r$ from the origin for us to solve the electric potential given by:
$$
begin{aligned}
V=kfrac{q}{r}
end{aligned}
$$
where $k=8.99times10^{9}text{N}cdottext{m}^2/text{C}^2$. To obtain $r$, we can use the distance formula:
$$
begin{aligned}
r=sqrt{(x_2-x_1)^2+(y_2-y_1)^2}
end{aligned}
$$
Step 2
2 of 8
For charge $q_1$ at (4.40 m, 6.22 m), the distance $r$ from the origin would be:
$$
begin{aligned}
r_1=sqrt{(4.40;text{m}-0;text{m})^2+(6.22;text{m}-0;text{m})^2}=7.62;text{m}
end{aligned}
$$
Step 3
3 of 8
For charge $q_2$ at (-4.50 m, 6.75 m), the distance $r$ from the origin would be:
$$
begin{aligned}
r_2=sqrt{(-4.50;text{m}-0;text{m})^2+(6.75;text{m}-0;text{m})^2}=8.11;text{m}
end{aligned}
$$
Step 4
4 of 8
Lastly, for charge $q_3$ at (2.23 m, -3.31 m), the distance $r$ from the origin would be:
$$
begin{aligned}
r_3=sqrt{(2.23;text{m}-0;text{m})^2+(-3.31;text{m}-0;text{m})^2}=3.99;text{m}
end{aligned}
$$
Step 5
5 of 8
Note that the total electric potential $V$ due to a number of charges is just equal to the algebraic sum of the electric potentials due to individual charges.
$$
begin{aligned}
V=V_1 + V_2 + V_3 = kfrac{q_1}{r_1}+kfrac{q_2}{r_2}+kfrac{q_3}{r_3}
end{aligned}
$$
Step 6
6 of 8
Since we need the potential at the origin to be zero, we let $V$ be equal to 0. Moreover, we can rearrange the equation such that we get the expression for $q_3$.
$$
begin{aligned}
0&=kfrac{q_1}{r_1}+kfrac{q_2}{r_2}+kfrac{q_3}{r_3}\
0&=frac{q_1}{r_1}+frac{q_2}{r_2}+frac{q_3}{r_3}\
frac{q_3}{r_3}&=-left(frac{q_1}{r_1}+frac{q_2}{r_2} right)\
q_3&=-r_3left(frac{q_1}{r_1}+frac{q_2}{r_2} right)
end{aligned}
$$
Step 7
7 of 8
We can now substitute our obtained values earlier and get charge $q_3$
$$
begin{aligned}
q_3&=-(3.99;text{m})left(frac{24.5times 10 ^{-6}text{C}}{7.62;text{m}}+frac{-11.2times 10 ^{-6}text{C}}{8.11;text{m}} right)\&=boxed{-7.32;mutext{C}}
end{aligned}
$$
Result
8 of 8
$q_3=-7.32;mutext{C}$
Exercise 119
Step 1
1 of 2
The direction of the electric field goes away from a positive charge and toward a negative charge. Since an electric eel produces a positive charge near its head and a negative charge near its tail, the direction of the electric field would point **towards the tail.**
Result
2 of 2
B.
Exercise 120
Step 1
1 of 3
The equation for the magnitude of charge $Q$ on either plate is given by:
$$
begin{aligned}
Q=CV
end{aligned}
$$
where $C$ is the capacitance of the capacitor and $V$ is the electric potential difference.
Step 2
2 of 3
Since we already know the needed values, $C=7.6;text{pF}$ and $V=650;text{V}$, we can substitute these values to the equation to obtain $Q$.
$$
begin{aligned}
Q&=(7.6times10^{-12};text{F})(650;text{V})\&=boxed{4.9times10^{-9};text{C}}
end{aligned}
$$
Result
3 of 3
C.
Exercise 121
Step 1
1 of 3
We treat the electric eel as a parallel-plate capacitor. The corresponding total energy $PE$ stored in a capacitor with charge $Q$ and potential difference $V$ is given by:
$$
begin{aligned}
PE=frac{1}{2}QV
end{aligned}
$$
Step 2
2 of 3
From Problem 120, the charge $Q$ has a value of $4.9times10^{-9};text{C}$ and a potential difference $V$ of $650;text{V}$. Substituting these values to the equation, we get:
$$
begin{aligned}
PE&=frac{1}{2}(4.9times10^{-9};text{C})(650;text{V})\
&=boxed{1.6times10^{-6};text{J}}
end{aligned}
$$
Result
3 of 3
C.
unlock
Get an explanation on any task
Get unstuck with the help of our AI assistant in seconds
New